Oct 21

设$f_{n}:\mathbf{R}^{n}\to \mathbf{R}$为连续函数,且
\[ \lim_{n\to\infty}f_{n}(x)=f(x)\qquad \forall x\in \mathbf{R}^{n} \]
则$f$的连续点集在$\mathbf{R}^{n}$中为稠密的$G_{\delta}$集.

————————————————————————————————————————

证明:

我们首先注意到,若$G\subseteq \mathbf{R}$为开集,则$f^{-1}(G)$为$F_{\sigma}$集
事实上,由于$R$中的开集$G$是可数个构成区间的并,故不妨设$G$是一个开区间$(a,b)$.而我们知道
\[ \{x|f_{k}(x)\geq a+\varepsilon\}  \]
是闭集,由于$f_{k}(x)$的连续性,从而
\[ \{x|f(x)>a \}=\bigcup_{n=1}^{\infty}\bigcup_{m=1}^{\infty}\bigcap_{k=m}^{\infty}\left\{x|f_{k}(x)\geq a+\frac{1}{n}\right\} \]
是$F_{\sigma}$集.同理$\{x|f(x)<b\}$也是$F_{\sigma}$集合.而他们的交集
\[ f^{-1}((a,b))=\{x|f(x)>a \}\bigcap\{x|f(x)<b\} \]
也是$F_{\sigma}$集.为了证明$f(x)$的连续点是稠密的$G_{\delta}$集,我们只要证$f(x)$的不连续点是没有内点的$F_{\sigma}$集.记$D(f)$为$f(x)$的不连续点集合,任意的$a\in D(f)$,存在$p,q\in \mathbf{Q}$使得
\[ p<f(a)<q\]
\[ \Leftrightarrow\qquad a\in f^{-1}((p,q)) \]
\[ a\notin f^{-1}(R-(p,q)) \]
而存在点列$\{a_{n}\}$使得
\[ \lim_{n\to\infty}a_{n}=a,\qquad f(a_{n})\notin(p,q) \]
\[ \Leftrightarrow\qquad a_{n}\notin f^{-1}(p,q) \]
\[ \Leftrightarrow\qquad a_{n}\in f^{-1}(R-(p,q)), \lim_{n\to\infty}a_{n}=a \]
于是
\[ a\in \overline{f^{-1}(R-(p,q))}-f^{-1}(R-(p,q)) \]
所以
\[ D(f)=\bigcup_{p<q,\text{$p,q$为有理数}}[\overline{f^{-1}(R-(p,q))}-f^{-1}(R-(p,q))] \]
而$f^{-1}(R-(p,q))$为$G_{\delta}$集,故
\[ \overline{f^{-1}(R-(p,q))}-f^{-1}(R-(p,q)) \]
为$F_{\sigma}$集.并且$\overline{f^{-1}(R-(p,q))}-f^{-1}(R-(p,q))=\partial f^{-1}(R-(p,q))$是没内点的.它是可数个无内点的闭集的并,所以$D(f)$也是可数个无内点的闭集的并。由Baire定理,$D(f)$是无内点的$F_{\sigma}$集.所以$R^{n}\setminus D(f)$是稠密集合.

 

从这个结论可以看出,导函数的连续点也是稠密的,只要注意到

\[ f'(x)=\lim_{n\to\infty}\frac{f\left(x+\frac{1}{n}\right)-f(x)}{\frac{1}{n}} \]

就好.
 

Jun 12

设$a_{0}$和$a_{1}$是实数,且满足$a_{n+1}=a_{n}+\frac{2}{n+1}a_{n-1}$,证明序列$\left\{\frac{a_{n}}{n^2}\right\}$收敛,并求极限。


解:设
\[ S(x)=a_{0}+a_{1}x+\cdots+a_{n}x^{n}+\cdots \]

\begin{align*}
S(x)&=a_{0}+a_{1}x+\sum_{n=2}^{\infty}a_{n}x^{n}\\
&=a_{0}+a_{1}x+\sum_{n=1}^{\infty}a_{n+1}x^{n+1}\\
&=a_{0}+a_{1}x+\sum_{n=1}^{\infty}\left(a_{n}x^{n+1}+\frac{2}{n+1}a_{n-1}x^{n+1}\right)\\
&=a_{0}+a_{1}x+x(S(x)-a_{0})+2\int_{0}^{x}tS(t)dt
\end{align*}
两边对$x$求导,得到微分方程
\[ (x-1)S'(x)+(2x+1)S(x)+a_{1}-a_{0}=0 \]
注意到初值$S_{0}=a_{0}$,解这个ODE,得到
\[ S(x)=\frac{1}{4}\cdot\left[\frac{(2x^2-6x+5)(a_{0}-a_{1})+(5a_{1}-9a_{0})e^{-2x}}{(x-1)^3}\right]\]
我们有展开式
\[ \frac{1}{(1-x)^{3}}=\sum_{k=0}^{\infty}\frac{(k+2)(k+1)}{2}x^{k} \]
\[ e^{-2x}=\sum_{k=0}^{\infty}\frac{(-2)^{k}}{k!}x^{k} \]

\[ (2x^2-6x+5)\cdot\frac{1}{(1-x)^3}=5+9x+\sum_{n=2}^{\infty}\frac{1}{2}(n+5)(n+2)x^{n} \]
\[ e^{-2x}\cdot\frac{1}{(1-x)^3}=\sum_{n=0}^{\infty}c_{n}x^{n} \]
其中
\[ c_{n}=\sum_{k=0}^{n}\frac{(-2)^{k}(n-k+2)(n-k+1)}{2\cdot k!} \]
于是
\[ S(x)=\frac{1}{4}(a_{1}-a_{0})\left(5+9x+\sum_{n=2}^{\infty}\frac{1}{2}(n+5)(n+2)x^{n}\right)+\left(\frac{9}{4}a_{0}-\frac{5}{4}a_{1} \right)\left(\sum_{n=0}^{\infty}c_{n}x^{n}\right)\]
对比$x^{n}$项的系数,得到
\[a_{n}=\frac{1}{8}(n+5)(n+2)(a_{1}-a_{0})+\left(\frac{9}{4}a_{0}-\frac{5}{4}a_{1}\right)\left(\sum_{k=0}^{n}\frac{(-2)^{k}(n-k+2)(n-k+1)}{2\cdot k!}\right)\]
下面来计算
\[ \lim_{n\to\infty}\frac{a_{n}}{n^2} \]
显然有
\[ \lim_{n\to\infty}\frac{\frac{1}{8}(n+5)(n+2)(a_{1}-a_{0})}{n^2}=\frac{1}{8}(a_{1}-a_{0}) \]
又注意到
\begin{align*}
&\frac{1}{n^2}\sum_{k=0}^{n}\frac{(-2)^{k}}{2\cdot k!}(n-k+2)(n-k+1)\\
&= \frac{1}{n^2}\sum_{k=0}^{n}\frac{(-2)^{k}}{2\cdot k!}[n^2+3n-2kn+(k-2)(k-1)]\\
&=\sum_{k=0}^{n}\frac{(-2)^{k}}{2\cdot k!}+o(1)\\
&\to \frac{e^{-2}}{2}\qquad (n\to\infty)
 \end{align*}
 所以
 \[ \lim_{n\to\infty}\frac{a_{n}}{n^2}=\left(\frac{9}{8}e^{-2}-\frac{1}{8}\right)a_{0}+\left(\frac{1}{8}-\frac{5}{8}e^{-2}\right)a_{1} \]

Jun 10

设$P_{0}(x)=\sqrt{x}$,$P_{n+1}(x)=\frac{1}{2}P_{n}^{2}(x)+(1-\sqrt{x})P_{n}(x)$,证明
\[ P_{n}(x)\rightrightarrows 0\qquad (x\in[0,1]) \]

证明:这个题源自周民强的习题集,实际是越南1989年的一个竞赛题的改编,但解答几乎都是采用了数学归纳法,外表看似合理,但是却难以想到。昨天看见西哥给出了不是归纳的办法,赞一个。现收录如下

不难发现$P_{n}(x)\geq P_{n+1}(x)$,和

\[ 0\leq P_{n}(x)\leq \sqrt{x} \]

现在注意到

\[ P_{n+1}(x)=P_{n}(x)\left(\frac{1}{2}P_{n}(x)+1-\sqrt{x}\right)\leq P_{n}(x)\left(1-\frac{1}{2}\sqrt{x}\right)\]

就有

\[ P_{n+1}(x)\leq P_{0}(x)\left(1-\frac{1}{2}\sqrt{x}\right)^{n}=\left(\frac{n}{2}\right)^{n}\sqrt{x}\left(\frac{2}{n}-\frac{1}{n}\sqrt{x}\right)^{n}\leq \left(\frac{n}{2}\right)^{n}\left(\frac{2}{n+1}\right)^{n+1}\leq \frac{2}{n+1} \]

显然就有

\[ P_{n}(x)\rightrightarrows 0\qquad (x\in[0,1]) \]

Jun 10

设$f(x)$在$[0,\pi]$上可微,且满足$(f'(x))^2 \in R[a,b]$,若$\displaystyle \int_{0}^{\pi}f(x)dx=0$,则
\[ \int_{0}^{\pi}f^{2}(x)dx\leq \int_{0}^{\pi}(f'(x))^2 dx \]


证明:我们把$f(x)$以$\pi$为周期先偶延拓到$[-\pi,\pi]$,再以$2\pi$为周期延拓到$\mathbb{R}$,这时$2k\pi,k\in \mathbf{N}^{+}$是$f(x)$的周期,注意到
\[ f(x+2\pi)=f(x),f(x)=f(-x) \]
两边对$x$求导,得到
\[ f'(x+2\pi)=f'(x),f'(x)=-f'(-x) \]
就是说$2k\pi,k\in \mathbf{N}^{+}$也是$f'(x)$的周期,而
\[ \int_{0}^{\pi}f^{2}(x)dx=\int_{-\pi}^{0}f^{2}(x)dx=\int_{\pi}^{2\pi}f^2(x)dx \]

\[ \int_{0}^{\pi}(f'(x))^2dx=\int_{-\pi}^{0}(f'(x))^2dx=\int_{\pi}^{2\pi}(f'(x))^2dx \]
不等式显然等价于
\[ \int_{0}^{2\pi}f^{2}(x)dx\leq \int_{0}^{2\pi}(f'(x))^2dx \]
设$f(0)=f(2\pi)=a$,这时,注意到
\[ 0\leq (f(x)-a)^2\cot{x}\leq \left(\int_{0}^{x}f'(t)dt\right)^{2}\frac{1}{\sin{x}}\leq \int_{0}^{x}dt\int_{0}^{x}(f'(t))^{2}dt\frac{1}{\sin{x}}=\frac{x}{\sin{x}}\int_{0}^{x}(f'(t))^{2}dt\to 0 (x\to 0^{+}) \]

\[ \lim_{x\to 0}(f(x)-a)^{2}\cot x=0 \]
同理也有
\[ \lim_{x\to 2\pi}(f(x)-a)^{2}\cot x=0 \]
不妨延拓定义
\[ (f(x)-a)^{2}\cot x\bigg|_{x=0}=(f(x)-a)^{2}\cot x\bigg|_{x=2\pi}=0 \]
现在,我们有
\begin{align*}
&\int_{0}^{2\pi}\bigg((f'(x))^2-(f(x)-a)^2-[f'(x)-(f(x)-a)\cot x]^{2}\bigg)dx\\
&=\int_{0}^{2\pi}\bigg(-(f(x)-a)^2\csc^{2}x+2f'(x)(f(x)-a)\cot x\bigg)dx\\
&=(f(x)-a)^{2}\cot x\bigg|_{0}^{2\pi}\\
&=0
\end{align*}

\[ \int_{0}^{2\pi}[(f'(x))^2-(f(x)-a)^2]dx=\int_{0}^{2\pi}[f'(x)-(f(x)-a)\cot x]^{2}dx\geq 0 \]
而注意到
\[ \int_{0}^{2\pi}f(x)dx=0 \]则
\[ \int_{0}^{2\pi}[(f'(x))^2-(f(x)-a)^2]dx=\int_{0}^{2\pi}((f'(x))^2-(f(x))^2)dx-2\pi a^2\geq 0 \]
就是说
\[ \int_{0}^{2\pi}(f'(x))^2dx\geq \int_{0}^{2\pi}(f(x))^2dx+2\pi a^2\geq \int_{0}^{2\pi}(f(x))^2dx \]
Hence we are done!

Jun 9

设$f(x)$是$[a,b]$上的单调函数,$\displaystyle F(x)=\int_{a}^{x}f(t)dt $在$[a,b]$上可导,则$f\in C[a,b]$.

——————————————————————————————————————————————————
证明:不妨设$f$单调递增,由于单调的$f$必然是可积函数,所以它的不连续点集是零测集,若$f$ 在$x_{0}$处连续,则有
\[ \lim_{x\to x_{0}}f(x)=f(x_0) \]
就是对任意$\varepsilon>0$,存在$\delta>0$,对任意的$ |h|<\delta$有
\[ |f(x_{0}+h)-f(x_{0})|<\varepsilon \]

于是
\[ \left|\frac{F(x_0+h)-F(x_0)}{h}-f(x_{0})\right|\leq \frac{1}{h}\int_{x_{0}}^{x_{0}+h}|f(t)-f(x_{0})|dt\leq \varepsilon \]
这说明
\[ F'(x_0)=f(x_0) \]
\[ m(\{x\in[a,b]|F'(x)\neq f(x)\})=0 \]
不难看到集合$A=\{x\in[a,b]|F'(x)=f(x)\}$是稠密集,现证明相反的方面,若$f$在$x_{0}$不连续,由于$f$单调,则有
\[ M=\lim_{x\to x_{0}^{+}}f(x)>\lim_{x\to x_{0}^{-}}f(x)=m \]
就是存在数列$x_{n}> x_{0},x_{n}\in A,x_{n}\to x_{0} (n\to+\infty)$,以及$y_{n}< x_{0},y_{n}\in A,y_{n}\to x_{0} (n\to+\infty)$
使得
\[ \lim_{n\to+\infty}f(x_{n})=M,\lim_{n\to+\infty}f(y_{n})=m \]
而注意到$x_{n},y_{n}$都是从集合$A$中选择的,所以又有
\[ f(x_{n})=F'(x_{n}) \]
\[ f(y_{n})=F'(y_{n}) \]
于是
\[ \lim_{n\to+\infty}F'(y_{n})<\lim_{n\to+\infty}F'(x_n) \]
注意到,若$x<y$,
则当$h$充分小时
\[ \frac{1}{h}\int_{x}^{x+h}f(t)dt\leq f(x+h)\leq f(y-h)\leq \frac{1}{h}\int_{y-h}^{y}f(t)dt \]
这就有
\[ F'(x)\leq F'(y) \]
于是$F'(x)$单调,就有
\[ \lim_{x\to x_{0}^{+}}F'(x)=\lim_{n\to+\infty}F'(x_n)>\lim_{n\to+\infty}F'(y_n)= \lim_{x\to x_{0}^{-}}F'(x) \]
这与$F'(x)$的介值性矛盾,所以必然有$f\in C[a,b]$。